OB: Chapter 14 Nursing Management During Labor and Birth, Chapter 14, Exam 1 prep- Chap 1-3

Réussis tes devoirs et examens dès maintenant avec Quizwiz!

active labor intensity

50-80 mmHg

Psychological response

Clear, calm info Time - don't rush it Support Confidence Control - who's in the room How prepared is she?

How does a woman who feels in control of the situation during labor influence her pain?

Feelings of control are inversely related to the client's report of pain.

What is an early deceleration?

a deceleration that declines as the contractions starts but returns to Baseline before the contraction is over.

A woman received morphine during labor to help with pain control. Which finding would the nurse need to monitor the newborn for after birth?

decreased alertness Morphine is a commonly used opioid for the management of pain during labor. It is associated with newborn respiratory depression, decreased alertness, inhibited sucking, and a delay in effective feeding. pg 473

A client who requested "no drugs" in labor asks the nurse what other options are available for pain relief. The nurse reviews several options for nonpharmacologic pain relief, and the client thinks effleurage may help her manage the pain. This indicates that the nurse will:

instruct the client or her partner to perform light fingertip repetitive abdominal massage. The relaxation technique of visualization is used in hypnobirthing or focused meditation. Controlled chest breathing is a technique used in Lamaze breathing. Pressing on trigger points is an acupressure technique. pg 465

A client has asked that an opioid be kept on standby in case she needs it for pain control. As a precaution, the nurse will also have which of medication readily available to reverse the effects of that opioid?

naloxone

Labor

rhythmic, regular contractions (5 min) with cervical change Heart rate increases 10-20bmp, RR increase CO increases 12-31% first stage, 50% second stage BP increases up to 35% WBC count increases as muscle is getting damaged

The nurse assesses her client and notes that the fetus is at +1 station. The nurse interprets +1 station as indicating that the fetal presenting part is at

1cm below the ischial spine

salt

An ionic compound made from the neutralization of an acid with a base.

The nurse is monitoring a client who has given birth and is now bonding with her infant. Which finding should the nurse prioritize and report immediately for intervention?

Maternal tachycardia and dropping blood pressure

General anesthesia is not used frequently in obstetrics because of the risks involved. There are physiologic changes that occur during pregnancy that make the risks of general anesthesia higher than it is in the general population. What is one of those risks?

Neonatal depression is possible.

The nurse caring for a client in preterm labor observes abnormal fetal heart rate (FHR) patterns. Which nursing intervention should the nurse perform next?

administration of oxygen by mask

Baseline FHR

average FHR range of 110-160 beats/min at term as assessed during a 10-minute period that excludes periodic and episodic changes and periods of marked variability Excludes accels/decels

third maneuver

determines presentation palpate area above pubis symphisis

A woman is in the fourth stage of labor. During the first hour of this stage, the nurse would assess the woman's fundus at which frequency?

every 15 minutes During the first hour of the fourth stage of labor, the nurse would assess the woman's fundus every 15 minutes and then every 30 minutes for the next hour. pg 491

A woman is in the fourth stage of labor. During the first hour of this stage, the nurse would assess the woman's fundus at which frequency?

every 15min

If a fetus were not receiving enough oxygen during labor because of uteroplacental insufficiency, which pattern would the nurse anticipate seeing on the monitor?

fetal heart rate declining late with contractions and remaining depressed

Hydroxyzine (Vistaril)

*50-100 mg IM* Does not relieve pain but reduces anxiety and potentiates opioid analgesic effects; can not be given IV is used to decrease N/V

Vaginal fluid

Acidic

The nurse instructs the client about skin massage and the gate-control theory of pain. Which statement would be appropriate for the nurse to include for client understanding of the nonpharmacologic pain relief methods?

This is a technique to prevent the painful stimuli from entering the brain. Gate control diverts the pain stimuli from the pain site by replacing with a comfort stimuli in a new location. pg 465

The levels of structural organization and body system

Chemical Cellular Tissue Organ System Organismal

The student nurse is preparing to assess the fetal heart rate (FHR). She has determined that the fetal back is located toward the client's left side, the small parts toward the right side, and there is a vertex (occiput) presentation. The nurse should initially begin auscultation of the fetal heart rate in the mother's:

left lower quadrant.

What is a FHR deceleration?

transient fall in the FHR

Normal color of amniotic fluid

clear

Acid

any compound that forms H+ ions in solution

Effacement

*0%: Cervical canal is 2cm long *50%: Cervical canal is 1cm long *100%: Cervical canal is obliterated

Dilation

*0cm: external cervical os is closed *5cm: external cervical os is halfway dilated *10cm: external cervical os is fully dilated and ready for birth passage.

A low-risk client is in the active phase of labor. The nurse evaluates the fetal monitor strip at 10:00 a.m. and notes the following: moderate variability, FHR in the 130s, occasional accelerations, and no decelerations. At what time should the nurse reevaluate the FHR?

10:30 a.m.

Base

A substance that decreases the hydrogen ion concentration in a solution.

The nurse is assessing a laboring client and notes: 5 cm dilated, 80% effaced, zero station, contractions every 2 to 3 minutes, lasting 50 seconds, becoming increasingly uncomfortable, and apprehensive but appropriate and focused on breathing and relaxation. The nurse determines which nursing diagnosis is most appropriate for this client?

Acute pain related to uterine contractions

A pregnant patient received a narcotic analgesic 2 hours before delivery. The newborn is lethargic and difficult to arouse. What should the nurse prepare to do to help this newborn?

Administer naloxone hydrochloride.

A client in labor has requested the administration of narcotics to reduce pain. At 2 cm cervical dilatation, she says that she is managing the pain well at this point but does not want it to get ahead of her. What should the nurse do?

Advise the client to hold out a bit longer, if possible, before administration of the drug, to prevent slowing labor.

A client has been in labor for 10 hours and is 6 cm dilated. She has already expressed a desire to use nonpharmacologic pain management techniques. For the past hour, she has been lying in bed with her doula rubbing her back. Now, she has begun to moan loudly, grit her teeth, and bear down with each contraction. She rates her pain as 8 out of 10 with each contraction. What should the nurse do first?

Assess for labor progression.

A nurse notes the digital readings of the electronic fetal monitor shows decreased beat-to-beat variability in a client who was just admitted to the unit. The nurse interprets this as indicating which system is mainly being affected in the fetus?

Central nervous system

A gravida 1 client is admitted in the active phase of stage 1 labor with the fetus in the LOA position. The nurse anticipates noting which finding when the membranes rupture?

Clear to straw-colored

A patient in labor is prescribed transcutaneous electrical nerve stimulation (TENS) to help with pain relief during labor. How should the nurse explain the process of pain relief with this method?

Counterirritation stimulation blocks pain from traveling to the spinal cord.

Cephalic presentation

Desire Vertex

Causes of fetal tachycardia

Early fetal hypoxia, maternal fever, maternal dehydration, beta-sympathomimetic drugs, amnionitis, maternal hyperthyroidism, fetal anemia

While assessing the progress of the labor, the nurse explains that the fetal heart rate variability is moderate. Which explanation is best to use with the parents?

FHR fluctuates from 6-25 beats per minute.

The nursing instructor is teaching a group of nursing students about the uniqueness of pain involved with the birthing process. The instructor determines the session is successful when the students correctly choose which pain factor to be related to psychosocial influences?

Fear of pain during labor

How does a woman who feels in control of the situation during labor influence her pain?

Feelings of control are inversely related to the client's report of pain. Studies reveal that women who feel in control of their situation are apt to report less pain than those who feel they have no control. pg 475

Strong Intensity

Feels like the forehead

Mild Intensity

Feels like the tip of the nose.

Breech presentation

Frank breech OK

Assessment of Contractions

Frequency Duration Intensity Uterine resting tone.

The nurse is caring for a client who appears tense and apprehensive as labor progresses. Which nursing intervention is most helpful?

Initiate comfort measures

Fentanyl has been administered to a client in labor. What assessment should the nurse prioritize?

Respiratory status

A nurse is caring for a client who has been administered an epidural block. Which should the nurse assess next?

Resting Respirations

Organelles

Structures specialized to perform distinct processes within a cell.

While waiting for the placenta to deliver during the third stage of labor the nurse must assess the new mother's vital signs every 15 minutes. What sign would indicate impending shock?

Tachycardia and a falling blood pressure

What is FHR variability?

The beat-to-beat fluctuations in FHR that are irregular in amplitude and frequency

Which assessment finding is most important as labor progresses?

The uterus relaxes completely between contractions.

The pain of labor is influenced by many factors. What is one of these factors?

The woman is prepared for labor and birth.

If the monitor pattern of uteroplacental insufficiency were present, which action would the nurse do first?

Turn her or ask her to turn to her side. The most common cause of uteroplacental insufficiency is compression of the vena cava; turning the woman to her side removes the compression. pg 463

At what time is the laboring client encouraged to push?

When the cervix is fully dilated

Preparation

Woman needs childbirth classes to help them cope with pain. Patients need to acquire skills to cope with labor discomfort. Don't have patient eat big meals, gastric motility and food absorption decrease --> she'll vomit

The nurse caring for a client in preterm labor observes abnormal fetal heart rate (FHR) patterns. Which nursing intervention should the nurse perform next?

administration of oxygen by mask The client should be administered oxygen by mask because the abnormal FHR pattern could be due to inadequate oxygen reserves in the fetus. Because the client is in preterm labor, it is not advisable to apply vibroacoustic stimulation, tactile stimulation, or fetal scalp stimulation.

A client in the latent phase of the first stage of labor is noted to be uncomfortable with intact membranes and mild contractions on assessment. The nurse should encourage the client to pursue which action?

ambulation ad lib

in a breech position the fetal heart rate is best heard

at or above the level of the maternal umbilicus

The nurse determines that the fetal heart rate averages approximately 140 beats per minute over a 10-minute period. The nurse identifies this as:

baseline FHR.

A nurse is caring for woman in labor. The woman's membranes just ruptured. The nurse assesses the characteristics of the fluid. Which finding would the nurse identify as normal?

clear

The nurse is providing preoperative care for a client who will undergo a cesarean section. The nurse should

confirm that consent has been provided by the client.

The nurse is providing preoperative care for a client who will undergo a cesarean section. The nurse should:

confirm that consent has been provided by the client.

A woman is experiencing back labor and reports intense pain in the lower back. Which is the most effective nursing intervention to relieve this type of pain?

counter pressure against the sacrum Counter pressure against the sacrum is a way to provide support and comfort for a women having intense back labor. Effleurage is ineffective for true back labor, as is conscious relaxation. Breathing will not diminish the pain of back labor. pg 483

DNA

deoxyribonucleic acid, a self-replicating material present in nearly all living organisms as the main constituent of chromosomes. It is the carrier of genetic information.

A 39-week-gestation client presents to the labor and birth unit reporting abdominal pain. What should the nurse do first?

determine if the client is in true or false labor.

What are Leopold maneuvers for?

determining the presentation, position, and lie of the fetus through the use of four specific steps

A woman is lightly stroking her abdomen in rhythm with her breathing during contractions. The nurse identifies this technique as:

effleurage

Which intervention would be least effective in caring for a woman who is in the transition phase of labor?

encouraging the woman to ambulate

A nurse practitioner is conducting an in-service education program for a group of nurses working in the labor and birth unit. The program is focusing on interpreting FHR patterns. The nurse practitioner determines that the teaching was successful when the group identifies which patterns as indicating abnormal fetal acid-base status? Select all that apply.

fetal bradycardia sinusoidal pattern recurrent late decelerations

The nurse is monitoring a client's uterine contractions. Which factors should the nurse assess to monitor uterine contraction? Select all that apply.

intensity of contractions frequency of contractions uterine resting tone The nurse should assess the frequency of contractions, intensity of contractions, and uterine resting tone to monitor uterine contractions. Monitoring changes in temperature and blood pressure is part of the general physical examination and does not help to monitor uterine contraction.

A client has presented in the early phase of labor, experiencing abdominal pain and signs of growing anxiety about the pain. Which pain management technique should the nurse prioritize at this stage?

practicing effleurage on the abdomen

uterine contraction

resembles a wave, moving downward to cervix and upward towards uterus

A nurse is monitoring a female client with an epidural block. Which complication would be the most important for the nurse to monitor in the client?

respiratory depression Respiratory depression is a complication of epidural anesthesia and should be closely monitored in laboring clients. A failed block, accidental intrathecal block, and a postdural headache are all side effects of a spinal epidural block.

A nurse is caring for a client who has been administered an epidural block. Which should the nurse assess next?

respiratory rate

What is a nursing intervention that helps prevent the most frequent side effect from epidural anesthesia in a pregnant client?

starting an IV and hanging IV fluids

protein synthesis

the formation of proteins by using information contained in DNA and carried by mRNA

A nursing student is studying labor and delivery and has learned that the first stage of labor consists of which of the following phases? (Select all that apply.)

transition active latent The first stage of labor includes three phases: latent, active, and transition.

t or f: During intermittent auscultation to establish a baseline, the FHR is assessed for a full minute after a contraction. From then on, unless there is a problem, listening for 30 seconds and multiplying the value by two is sufficient.

true

While caring for a woman in labor, the nurse notes that the fetal heart monitor demonstrates late decelerations. The most common cause for their occurrence is:

uteroplacental insufficiency. Late decelerations are associated with uteroplacental insufficiency. They typically indicate decreased blood flow to the uterus during the contractions. Maternal hypotension and fatigue would not be observed on the fetal heart monitor. Cord compression would be marked by fetal tachycardia.

Adverse effects of opioids

• nausea and vomiting • drowsiness • Itching • constipation RESPIRATORY DEPRESSION

Fentanyl (Sublimaze)

*50-100mcg IV Or Epidurally Can cause maternal Hypotension, Maternal and fetal respiratory depression. Rapidly crosses the placenta.

Sources of Discomfort

*Tissue Ischemia (tissue hypoxia, uterus) *Cervical Dilation *Pelvic Structures *Distention of the Vagina

Amniotic Fluid

Alkaline. Turns Nitrate Swab blue. If they have no ruptured the swab remains yellow to olive green.

Nalbuphine (Nubain)

*10-20mg IV* Causes less maternal nausea and vomiting. Causes decreased FHR variability, fetal bradycardia, and respiratory depression.

Morphine

*2-5mg IV* Rapidly crosses the placenta, causes a decrease in FHR variability. Can cause maternal and neonatal CNS depression. Decrease uterine contractions.

Promethazine (Phenergan)

*25-50mg IV* Is used for antiemetic effect when combined with opioids. Causes sedation and reduces apprehension. May contribute to maternal hypotension and neonatal depression.

A client has just received IV sedation. What must the nurse tell the client to do?

Ambulate only with assistance from the nurse or caregiver. The client may have decreased sensory ability from the medication. She needs assistance to ambulate for safety. She will be largely unable to move, so she should remain in bed unless absolutely necessary. pg 475

Protein

An organic compound that is made of one or more chains of amino acids and that is a principal component of all cells

A client has just received combined spinal epidural. Which nursing assessment should be performed first?

Assess vital signs. The most common side effect of spinal and epidural anesthesia is hypotension, which can lead to fetal bradycardia, decelerations, or fetal distress. Although each is important, this nursing assessment should be performed first.

Priority Focus When Membranes Rupture

Assessing the FHR first to identify deceleration. Which may indicate Cord compression secondary to cord prolapse.

Which primary symptom does the nurse identify as a potentially fatal complication of epidural or intrathecal anesthesia?

Difficulty breathing

Lipids

Energy-rich organic compounds, such as fats, oils, and waxes, that are made of carbon, hydrogen, and oxygen.

A pregnant client has opted for hydrotherapy for pain management during labor. Which measure should the nurse consider when assisting the client during the birthing process?

Initiate the technique only when the client is in active labor. The recommendation for initiating hydrotherapy is that women be in active labor (>5 cm dilated) to prevent the slowing of labor contractions secondary to muscular relaxation. Women are encouraged to stay in the bath or shower as long as they feel they are comfortable. The water temperature should not exceed body temperature. The woman's membranes can be intact or ruptured.

A multigravida is admitted to the hospital in active labor. The client's and the fetus's condition have been good since admission. The client calls out to the nurse, "the baby is coming!" What is the first action of the nurse?

Inspect the perineum. The nurse needs to determine if birth is imminent and be prepared for birth. Once the nurse assesses the coming labor, the heart sounds, contraction rate, and contacting the primary care provider can all be done, if there is time.

Nitrazine Swab

Taken from the vagina to determine the membranes have ruptured. It determines the ph. Can be false positive if too much bloody show due to presence of blood.

To assess the frequency of a woman's labor contractions, the nurse would time:

The beginning of one contraction to the beginning of the next

The client may spend the latent phase of the first stage of labor at home unless which occurs?

The client experiences a rupture of membranes

During labor, a pregnant patient's doula uses therapeutic touch and massage. Which outcome indicates that these approaches have been effective?

The patient is not requesting pain medication.

Purpose of Vaginal Examination

To assess the amount of cervical dilation, the percentage of cervical effacement and the fetal membrane status.

During an admission assessment of a client in labor, the nurse observes that there is no vaginal bleeding yet. What nursing intervention is appropriate in the absence of vaginal bleeding when the client is in the early stage of labor?

assess amount of cervical dilation

A woman is in the fourth stage of labor. During the first hour of this stage, the nurse would assess the woman's fundus at which frequency?

every 15 minutes During the first hour of the fourth stage of labor, the nurse would assess the woman's fundus every 15 minutes and then every 30 minutes for the next hour.

A client is being admitted to labor and birth. When admitting an obstetric client in early labor, the first intervention by the nurse is:

good rapport is established with the client and significant other. On admission the client and her family need to establish a rapport with their caregiver. If the client is stable and there is no immediate need, rapport should be established over actions that can be taken care of later.

Early in labor, a pregnant client asks why contractions hurt so much. Which answer should the nurse provide?

lack of oxygen to the muscle fibers of the uterus due to compression of blood vessels During contractions, blood vessels constrict, reducing the blood supply to uterine and cervical cells, resulting in anoxia to muscle fibers. This anoxia can cause pain in the same way blockage of the cardiac arteries causes the pain of a heart attack. Endorphins are naturally occurring opiate-like substances that reduce pain, not cause it. Distraction and mechanical irritation of nerve fibers are also methods of reducing pain, not causes of pain.

Branches of Physiology

neurophysiology, endocrinology, cardiovascular physiology, immunology, respiratory physiology, renal physiology, exercise physiology, pathophysiology

When assessing fetal heart rate patterns, which finding would alert the nurse to a possible problem?

prolonged decelerations

During labor, a pregnant patient's doula uses therapeutic touch and massage. Which outcome indicates that these approaches have been effective?

the patient is not requesting pain medication

Chemical bonds are

the redistribution of electrons between atoms, leading to a more stable configuration

What is a spinal block?

injection of a local agent into the subarachnoid space, similar to an epidural. Mainly used for scheduled c/s

A client has asked that an opioid be kept on standby in case she needs it for pain control. As a precaution, the nurse will also have which of medication readily available to reverse the effects of that opioid?

naloxone Naloxone is an opioid antagonist that can be given to reverse the effects of the central nervous system depression, including respiratory depression, caused by opioids. It is also used to reverse the side effects of neuraxial opioids. Nalbuphine is an opioid. Hydroxyzine is an ataractic. Midazolam is a benzodiazepine. pg 473

A client has presented in the early phase of labor. She is experiencing abdominal pain and shows signs of growing anxiety about the pain. What is the best pain management technique the nurse can suggest at this stage?

practicing effleurage on the abdomen In early labor, the less medication use the better; allow use of nonpharmacologic management and control the pain with gate theory. Sitting in a warm pool of water is relaxing and may lessen the pain, but it does not control the pain. Sedatives are not indicated as they may slow the birthing process. Opioids should be limited as they too may slow the progression of labor. pg 465

While waiting for the placenta to deliver during the third stage of labor the nurse must assess the new mother's vital signs every 15 minutes. What sign would indicate impending shock?

tachycardia and a falling blood pressure Monitor the woman's vital signs at least every 15 minutes during the third stage of labor. Tachycardia and a falling blood pressure are signs of impending shock; the nurse should immediately report these signs.

A client is in active labor. Checking the EFM tracing, the nurse notes variables that are abnormal. What would be the nurse's first nursing intervention?

Help the woman change positions

How can we assess FHR?

a fetoscope (a modified stethoscope attached to a headpiece) or a Doppler (ultrasound) device, or continuously with an electronic fetal monitor applied externally or internally

The nurse determines that the fetal heart rate averages approximately 140 beats per minute over a 10-minute period. The nurse identifies this as:

baseline FHR. The baseline FHR averages 110 to 160 beats per minute over a 10-minute period. Fetal bradycardia occurs when the FHR is less than 110 beats per minute for 10 minutes or longer. Short-term variability is the beat-to-beat change in FHR. Baseline variability refers to the normal physiologic variations in the time intervals that elapse between each fetal heartbeat observed along the baseline in the absence of contractions, decelerations, and accelerations.

The labor and delivery nurse knows that internal monitoring during labor is an invasive technique. She correctly identifies an increase in the risk of which of the following for both mother and fetus if this is done?

infection Because internal monitoring techniques are invasive, both the woman and the fetus can become ill with infection. Internal methods should be used only when the benefit clearly outweighs the risk.

A nurse is caring for a client who has just received an epidural. Which side effect is the most common in epidural anesthesia?

maternal hypotension, which can lead to fetal bradycardia Epidural anesthesia conveys the risk of hypotension, especially if the client has not received an adequate amount of fluid before the procedure is performed. A sudden drop in maternal blood pressure can cause uterine hypoperfusion, which may result in fetal bradycardia. pg 474

The nurse is reviewing the medication administration record (MAR) of a client at 39 weeks' gestation and notes that she is ordered an opioid for pain relief. Which is an assessment priority after administering?

Assess fetal heart rate.

Tonicity

The ability of a solution surrounding a cell to cause that cell to gain or lose water.

A G3 P2 with no apparent risk factors presents to the labor-and-delivery suite in early labor. She refuses the fetal monitor, stating she delivered her second baby at home without a monitor and everything went well. What is the nurse's best response?

A few minutes on the monitor will ensure the baby is doing well and then the baby can then be monitored intermittently.

During an admission assessment of a client in labor, the nurse observes that there is no vaginal bleeding yet. What nursing intervention is appropriate in the absence of vaginal bleeding when the client is in the early stage of labor?

Assess amount of cervical dilation. If vaginal bleeding is absent during admission assessment, the nurse should perform vaginal examination to assess the amount of cervical dilation. Hydration status is monitored as part of the physical examination. A urine specimen is obtained for urinalysis to obtain a baseline. Vital signs are monitored frequently throughout the maternal assessment.

During contractions, the electronic fetal monitor (EFM) shows variable V-shaped decelerations in the FHR lasting about 30 seconds with accelerations of about 5 bpm before and after each deceleration. Overshoot is absent, and the baseline FHR is within normal limits. What should the nurse do first?

Help the woman change positions. Changing positions is a first intervention to determine if this will improve the oxygen to the fetus. Supplemental oxygen should be maintained until the mother is stable. Placing the client on her side may increase the work of breathing. Pharmacological interventions are premature.

Fetal Attitude

The arrival at the outlet. Desire full flexion, but may also see transverse lie and longitudinal lie

Fetal station

The descent and arrangement entering the bony pelvis Use fetal head: fontanels to find placement 0 = when presenting at level of spines -1 = 1cm above ischial spines =2 = 2cm below ischial spines

A client received IV meperidine for pain. An hour later a full-term neonate is born with respiratory depression. The nurse anticipates the neonate will require administration of which medication?

naloxone Naloxone is the drug used for reversal of opioids' adverse effects. If a narcotic is given too close to birth, the fetal liver takes 2 to 3 hours to activate a drug, the effect will not be registered (respiratory depression, sleepiness) in the fetus for 2 to 3 hours after birth. Indomethacin is an analgesic and NSAID; ampicillin is an antibiotic; and epinephrine is a vasopressor. pg 471

Colloids

substances such as large protein molecules that do not readily dissolve into true solutions

Mitosis

part of eukaryotic cell division during which the cell nucleus divides

A woman in early labor is using a variety of techniques to cope with her pain. When the nurse enters the room she notes that the woman is making light, circling movements with her fingertips across her abdomen. What technique is she using?

effleurage Effleurage is a form of touch that involves light circular fingertip movements on the abdomen and is a technique the woman can use in early labor. The theory is that light touch stimulates the nerve pathways to the brain and keeps them busy, thereby blocking the pain sensation. pg 470

A client asks why she should learn breathing patterns for labor. After instruction is given, the nurse determines teaching has been effective when the client states:

"Breathing patterns are distraction techniques taught to decrease pain in labor." Breathing techniques are largely distraction techniques taught in birth classes and are well documented to decrease pain in labor. pg 471

A woman states that she does not want any medication for pain relief during labor. Her primary care provider has approved this for her. What the nurse's best response to her concerning this choice?

"I respect your preference whether it is to have medication or not." Individualizing care to meet women's specific needs is a nursing responsibility. pg 464

A woman's primary care provider has told her he wants to use an episiotomy for birth. She asks the nurse what the purpose of this is. Which answer would be best?

"It relieves pressure on the fetal head." An episiotomy widens the vaginal opening, decreasing pressure on the fetal head. pg 483

A woman who has just been admitted in labor indicates that her husband is parking the car and her doula is on her way. The nurse's best response would be:

"OK, let me ask you some personal questions before she arrives." Doulas are birth coaches who provide one-on-one support in labor and throughout birth. A nurse would want to ask the client any personal questions before the doula arrives because the primary purpose of the doula is to be a continuous presence. A doula does not take the place of a nurse.

Opioids are often used in labor for pharmacologic pain management. A client in the transition phase of labor is requesting fentanyl for pain. How should the nurse respond to her request?

"Pain medication given now might cause the baby to have slow respirations and is not recommended; let's try to focus and breathe." Once the woman has entered into the transition phase of labor, she is considered to be imminent for birth. Any opioid medication might pass to the fetus and is not recommended due to the effects of respiratory compromise. The nurse will need to encourage nonpharmacologic methods at this point and should not consult the provider. The nurse should also remain supportive of the mother.

The nurse is admitting a primigravida client who has just presented to the unit in early labor. Which response should the nurse prioritize to assist the client in remaining calm and cooperative during birth?

"The baby is coming. I'll explain what's happening and guide you."

An experienced nurse is mentoring a graduate nurse and critiquing the graduate's shift handoff. Which statement requires clarification?

"The client reports a pain level of 8. She has a low pain tolerance."

The nurse is assessing a new client who presents in early labor. The nurse determines the fetus has an acceptable heart rate if found within which range?

110 to 160 bpm

Stages of Labor

1: onset -> 10cm dilated 2: 10cm -> baby 3: baby -> placenta 4: placenta -> 2-4hrs PP, depending on facility

The nurse tests the pH of fluid found on the vaginal exam and determines that the woman's membranes have ruptured based on which result?

6.5

The nurse is caring for a client in active labor who has had a fetal blood sampling to check for fetal hypoxia. The nurse determines that the fetus has acidosis when the pH is:

7.15 or less. In the hypoxic fetus, the pH will fall below 7.2, which is indicative of fetal distress.

A client has been in labor for 10 hours and is 6 cm dilated. She has already expressed a desire to use nonpharmacologic pain management techniques. For the past hour, she has been lying in bed with her doula rubbing her back. Now, she has begun to moan loudly, grit her teeth, and bear down with each contraction. She rates her pain as 8 out of 10 with each contraction. What should the nurse do first?

Assess for labor progression. Performing breathing exercises, ambulating, changing position, and emptying the bladder all can help the client experience a reduction in pain. However, the best first step is to assess the client for labor progress before assisting her otherwise. Bearing down can be a sign that the client is 10 cm dilated. pg 451

Enzymes

Catalysts for chemical reactions in living things

A nurse is monitoring the FHR of a client in labor using an electronic fetal monitor. The reading shows a late deceleration. Which intervention should the nurse implement?

Change maternal position to an upright or side lying position. To intervene with late decelerations, the nurse should change maternal position to an upright or side lying posture. Late deceleration in the fetus indicates insufficient uteroplacental perfusion. Changing the maternal position improves the maternal venous return. In upright position, the uterine activity becomes more efficient. Attempts should be made to increase the uteroplacental perfusion and fetal circulation. Administering oxytocin and encouraging Valsalva maneuver (extended breath holding) may augment the uteroplacental insufficiency. In late deceleration, the nurse should administer oxygen through nasal cannula and discontinue administration of oxytocin. Placing the client in the lithotomy position contributes to poor placental circulation pg 463

The laboring client is on continuous fetal monitoring when the nurse notes a decrease in the fetal heart rate with variable deceleration to 75 bpm. What is the initial nursing intervention?

Change the position of the client. Variable decelerations often indicate a type of cord compression. The initial response is to change the position and try to release the cord compression. If this does not work, apply oxygen while using the call light to alert others. If this continues, her fluid status needs to be assessed before increasing her IV rate. pg 460

The nurse is assessing a woman in active labor. She notes a small mass above the symphysis pubis, rounded, distended, and nontender. What intervention should the nurse take next?

Check the chart for the last void. The most probable explanation of the mass is the bladder, which is full. The nurse should determine the last void by the client and offer to assist the client to void or prepare to catheterize the client to empty the bladder. This can be taken care of by the nurse. The client would not likely know if the mass was always present or not, given its location. If it were the uterus, it would be tender to the touch.

The nurse is assessing a woman in active labor. She notes a small mass above the symphysis pubis, rounded, distended, and nontender. What intervention should the nurse take next?

Check the chart for the last void. The most probable explanation of the mass is the bladder, which is full. The nurse should determine the last void by the client and offer to assist the client to void or prepare to catheterize the client to empty the bladder. This can be taken care of by the nurse. The client would not likely know if the mass was always present or not, given its location. If it were the uterus, it would be tender to the touch. pg 486

The nurse is preparing a birthing care plan for a pregnant client. Which factor should the nurse prioritize to achieve adequate pain relief during the birthing process?

Client priorities and preferences are incorporated into the plan.

A nurse is caring for a client administered general anesthesia for an emergency cesarean birth. The nurse notes the client's uterus is relaxed upon massage. What would the nurse do next?

Continue to massage the client's fundus. The nurse should monitor the client for uterine relaxation. If this is noted, the nurse would continually massage the client's fundus until it no longer felt boggy.

The nursing instructor is preparing a class discussing the role of the nurse during the labor and birthing process. Which intervention should the instructor point out has the greatest effect on relieving anxiety for the client?

Continuous labor support

The nurse is preparing a young couple for the upcoming birth of their child, and the mother expresses concern for needing pain medications and the effects on the fetus. When counseling the couple about pain relief, the nurse would incorporate which information in the teaching about measures to help to decrease the requests for pain medication?

Continuous support through the labor process helps decrease the need for pain medication. Continuous labor support involves offering a sustained presence to the laboring woman. A support person can assist and provide aid with acupressure, massage, music therapy, or therapeutic touch. Research has validated the value of continuous labor support versus intermittent support in terms of lower operative deliveries, cesarean births, and request for pain medication. pg 465

A nursing student is learning about intermittent fetal heart rate monitoring during labor. The student correctly chooses which of the following as used routinely for this procedure? (Select all that apply.)

Doppler fetoscope fetal monitor Intermittent fetal heart rate ascultation uses fetoscope, Doppler, or fetal monitor. An intrauterine pressure catheter is inserted into a pocket of amniotic fluid and is a continuous internal monitoring of contractions.

The client is in the second stage of labor and is at 2+ station and in anticipation of birth within the hour. Her epidural did not work, and she is begging for meperidine. Which is the most appropriate action by the nurse?

Encourage her through the contractions, and explain why she is not receiving the meperidine. At this point, any medication would be contraindicated as it would pass to the fetus and may cause respiratory depression. The nurse will have to work with the mother through the contractions and pushing. The client has progressed too far to retry the epidural medication. No meperidine should be given due to the risk to the fetus.

A nurse notes a pregnant woman has just entered the second stage of labor. Which interaction should the nurse prioritize at this time to assist the client?

Encourage the woman to push when a strong desire to do so is present.

A client presents to the birthing center in labor. The client's membranes have just ruptured. Which assessment is the nurse's priority?

FHR When membranes rupture, the priority focus should be on assessing fetal heart rate first to identify a deceleration, which might indicate cord compression secondary to cord prolapse. Prolonged rupture can lead to an infection. Assessing the fetal position and maternal comfort are important but should not be the primary focus.

Moderate Intensity

Feels like the chin

Normal Uterine Contractions

Have a contraction (systole) and relaxation (diastole) phase. Resembles a wave that moves downward to the cervix and upward to the fundus of the uterus. Puts enormous pressure on fetus, thins and dilates the cervix

Which of the following supports why a preterm fetus usually is more affected by medication given at birth than a full-term fetus?

Inability of the immature liver to metabolize or inactivate drugs

Prolonged Rupture of Membranes

Increases the risk for infection to the mother and fetus due to ascending vaginal organisms.

Green Fluid

Indicates the fetus has passed Meconium due to fetal hypoxia.

A nurse is teaching a couple about patterned breathing during their birth education. Which technique should the nurse suggest for slow-paced breathing?

Inhale slowly through nose and exhale through pursed lips.

A nurse is teaching a couple about patterned breathing during their birth education. Which technique should the nurse suggest for slow-paced breathing?

Inhale slowly through nose and exhale through pursed lips. For slow-paced breathing, the nurse should instruct the woman to inhale slowly through her nose and exhale through pursed lips. In shallow or modified-pace breathing, the woman should inhale and exhale through her mouth at a rate of 4 breaths every 5 seconds. In pattern-paced breathing, the breathing is punctuated every few breaths by a forceful exhalation through pursed lips. Holding the breath for 5 seconds after every three breaths is not recommended in any of the three levels of patterned breathing.

There are advantages and disadvantages to any kind of method used to control pain during labor and birth. What is an advantage of opioid administration?

It has the ability to be administered by the nurse Opioids are most frequently given by the intravenous (IV) route because this route provides fast onset and more consistent drug levels than do the subcutaneous or intramuscular routes.

Fetal lie

LIE: relationship to the pelvis - longitudinal or transverse

The student nurse is preparing to assess the fetal heart rate (FHR). She has determined that the fetal back is located toward the client's left side, the small parts toward the right side, and there is a vertex (occiput) presentation. The nurse should initially begin auscultation of the fetal heart rate in the mother's:

LLQ

Causes of fetal bradycardia

Late fetal hypoxia, maternal hypotension, prolonged umbilical cord compression, fetal arrhythmia, uterina hyperstimulation, abruptio placentae, uterine rupture, vagal stimulation, congenital heart block, maternal hypothermia

Local anesthetic we use during labor for specific areas?

Lidocaine

The nurse is admitting a client in early labor and notes: FHR 120 bpm, blood pressure 126/84 mm Hg, temperature 98.8oF, contractions every 4 to 5 minutes lasting 30 seconds, and greenish-color fluid in the vaginal vault. Which finding should the nurse prioritize?

Meconium in the fluid

General anesthesia is not used frequently in obstetrics because of the risks involved. There are physiologic changes that occur during pregnancy that make the risks of general anesthesia higher than it is in the general population. What is one of those risks?

Neonatal depression is possible. General anesthesia is not used frequently in obstetrics because of the risks involved. The pregnant woman is at higher risk for aspiration. It requires more skill to intubate a pregnant woman because of physiologic changes in the trachea and thorax. In addition, general anesthetic agents cross the placenta and can result in the birth of a severely depressed neonate who requires full resuscitation. pg 476

Fundus palpation for contraction intensity

Nose-mild contraction Chin-moderate contraction Forehead-strong contraction

Continuous Labor Support

Offering a sustained presence to the laboring woman by providing emotional support, comfort measures, advocacy, information and advice and support for the partner.

The nurse is caring for a client who is sent to the obstetric unit for evaluation of fetal well-being. At which location is the nurse correct to place the tocodynamometer?

On the uterine fundus

Chemical elements that make up proteins

Oxygen, carbon, hydrogen, nitrogen and sulfur

The nurse is preparing an educational event for pregnant women on the topic of labor pain and birth. The nurse understands the need to include the origin of labor pain for each stage of labor. What information will the nurse present for the first stage of labor?

Pain originates from the cervix and lower uterine segment.

A client is in active labor. As one of the nursing diagnoses is "Risk for trauma to the woman or fetus related to intrapartum complications or a full bladder," what would be appropriate for the nurse to do in order to achieve the goal of "no complications due to a full bladder"?

Palpate the area above the symphysis pubis every two hours. Another source of trauma that can interfere with the progress of labor is a full bladder. Every two hours the nurse should palpate the area just above the symphysis pubis feeling for a rounded area of distention, which indicates the bladder is full. pg 481

A nurse is auscultating the fetal heart rate of a woman in labor. To ensure that the nurse is assessing the FHR and not the mother's heart rate, which action would be most appropriate for the nurse to do?

Palpate the mother's radial pulse at the same time.

What is a late deceleration?

Starts after contraction Slow return to baseline Fetal hypoxia

The nurse is admitting a client who appears to be in advanced labor with imminent birth. Which action should the nurse prioritize?

Take blood pressure and determine if clonus or edema are present.

The laboring client who is at 3 cm dilation and 25% effaced is asking for analgesia. The nurse explains the analgesia usually is not administered prior to the establishment of the active phase. What is the appropriate rationale for this practice?

This may prolong labor and increase complications. Administration of pharmacologic agents too early in labor can stall the labor and lengthen the entire labor. The client should be offered nonpharmacologic options at this point until she is in active labor. pg 471

The nursing instructor is teaching the students the basics of the labor and delivery process. The instructor determines the session is successful when the students correctly choose which action will best help to prevent infections in their clients?

Thoroughly wash the hands before and after client contact.

If the monitor pattern of uteroplacental insufficiency were present, which action would the nurse do first?

Turn her or ask her to turn to her side. The most common cause of uteroplacental insufficiency is compression of the vena cava; turning the woman to her side removes the compression.

A nurse caring for a pregnant client in labor observes that the fetal heart rate (FHR) is below 110 beats per minute. Which interventions should the nurse perform? Select all that apply.

Turn the client on her left side. Administer oxygen by mask. Assess client for underlying causes.

A nurse caring for a pregnant client in labor observes that the fetal heart rate (FHR) is below 110 beats per minute. Which interventions should the nurse perform? Select all that apply.

Turn the client on her left side. Administer oxygen by mask. Assess client for underlying causes. The nurse should turn the client on her left side to increase placental perfusion, administer oxygen by mask to increase fetal oxygenation, and assess the client for any underlying contributing causes. The client's questions should not be ignored; instead, the client should be reassured that interventions are to effect FHR pattern change. A reduced IV rate would decrease intravascular volume, affecting the FHR further.

The nurse is caring for a client who prefers resting on her back during the labor process. To facilitate client wishes, which nursing action is required?

Utilize a wedge under one hip

What is VEAL CHOP?

Variable - Cord compression Early - Head Compression Acceleration - Okay Late - Placental insufficiency

What is a prolonged deceleration?

Visually apparent decrease in the FHR below baseline that lasts at least 2 minutes but less than 10.

What do we need to assess for the mother during labor?

Vital signs and a vaginal exam, along with health hx/prenatal record

There has been much research done on pain and the perception of pain. What is the result of research done on levels of satisfaction with the control of labor pain?

Women report higher levels of satisfaction when they felt they had a high degree of control over the pain experience. Research has shown that women report higher levels of satisfaction with their labor experience when they feel a high degree of control over the experience of pain (Stuebe & Barbieri, 2005). pg 464

Meiosis

a type of cell division that results in four daughter cells each with half the number of chromosomes of the parent cell, as in the production of gametes and plant spores.

The expected fetal heart rate response in an active fetus is:

acceleration of at least 15 bpm for 15 seconds. A reassuring active fetal heart rate is a change in baseline by increase of 15 bpm for 15 seconds. This is a positive and reassuring periodic change in fetal heart rates as a response to fetal movement.

The nurse is reviewing the uterine contraction pattern and identifies the peak intensity, documenting this as which phase of the contraction?

acme

The nurse is reviewing the uterine contraction pattern and identifies the peak intensity, documenting this as which phase of the contraction?

acme The acme is the peak intensity of a contraction. The increment refers to the building up of the contraction. The decrement refers to the letting down of the contraction. Diastole refers to the relaxation phase of a contraction pg 453

The nurse is reviewing the uterine contraction pattern and identifies the peak intensity, documenting this as which phase of the contraction?

acme The acme is the peak intensity of a contraction. The increment refers to the building up of the contraction. The decrement refers to the letting down of the contraction. Diastole refers to the relaxation phase of a contraction.

The coach of a client in labor is holding the client's hand and appears to be intentionally applying pressure to the space between the first finger and thumb on the back of the hand. The nurse recognizes this as which form of therapy?

acupressure

The coach of a client in labor is holding the client's hand and appears to be intentionally applying pressure to the space between the first finger and thumb on the back of the hand. The nurse recognizes this as which form of therapy?

acupressure Acupressure is the application of pressure or massage at designated susceptible body points. A common point used for a woman in labor is Co4, which is located between the first finger and thumb on the back of the hand. Women may report their contractions feel lighter when a support person holds and squeezes their hand because the support person is accidentally triggering this point. Acupuncture involves insertion of needles into the same body points. Effleurage, the technique of gentle abdominal massage often taught with Lamaze in preparation for birth classes is a classic example of therapeutic touch. Biofeedback is based on the belief people have control and can regulate internal events such as heart rate and pain responses. pg 465

The nurse is assessing a laboring client and notes: 5 cm dilated, 80% effaced, zero station, contractions every 2 to 3 minutes, lasting 50 seconds, becoming increasingly uncomfortable, and apprehensive but appropriate and focused on breathing and relaxation. The nurse determines which nursing diagnosis is most appropriate for this client?

acute pain related to uterine contractions

The nurse caring for a client in preterm labor observes abnormal fetal heart rate (FHR) patterns. Which nursing intervention should the nurse perform next?

admin of O2 by mask

The nurse caring for a client in preterm labor observes abnormal fetal heart rate (FHR) patterns. Which nursing intervention should the nurse perform next?

administration of O2 mask

The nurse caring for a client in preterm labor observes abnormal fetal heart rate (FHR) patterns. Which nursing intervention should the nurse perform next?

administration of oxygen by mask The client should be administered oxygen by mask because the abnormal FHR pattern could be due to inadequate oxygen reserves in the fetus. Because the client is in preterm labor, it is not advisable to apply vibroacoustic stimulation, tactile stimulation, or fetal scalp stimulation. pg 460

A client is in the first stage of labor, latent phase. Her membranes are intact, and her contractions are mild. Considering the client's condition and phase of labor, the nurse knows that which aid will facilitate labor?

ambulation ad lib To facilitate the first stage of labor, ambulation and movement will allow better fetal descent and help to speed the labor process. Bed rest will slow or stop the labor process. The client may use the bathroom as needed, but this does not affect labor rate. The client should remain mobile.

A primigravida client has just arrived in early labor and is showing signs of extreme anxiety over the birthing process. Why should the nurse prioritize helping the client relax?

anxiety can slow down labor and decrease oxygen to the fetus.

key assessment during membrane rupture

assessing fetal heart rate (FHR) first to identify a deceleration, which might indicate cord compression secondary to cord prolapse.

The client and her partner have prepared for a natural birth and bring a picture of a sunset over the ocean with them. The nurse predicts they will be using which technique during labor?

attention focusing

Chemical elements that make up lipids

carbon, hydrogen, oxygen

fourth maneuver

determines fetal attitude (flexion or extensions) use tips of 3 fingers to palpate the abdomen, finger tips facing downwards.

A client in the first stage of labor is admitted to a health care center. The nurse caring for the client instructs her to rock on a birth ball. The nurse informs her that this causes the release of certain natural substances, which reduces the pain. To which substance is the nurse referring?

endorphins The nurse is referring to the release of endorphins, which are natural analgesic substances released by the movement of the client on the birth ball. The nurse should encourage the client to rock or sit on the birth ball. This causes the release of endorphins. The client's movement on the birth ball does not produce prostaglandins, progesterone, or relaxin. Prostaglandins are local hormones that bring about smooth muscle contractions in the uterus. Progesterone is a hormone involved in maintaining pregnancy. Relaxin is a hormone that causes backache during pregnancy by acting on the pelvic joints.

A client in the first stage of labor is admitted to a health care center. The nurse caring for the client instructs her to rock on a birth ball. The nurse informs her that this causes the release of certain natural substances, which reduces the pain. To which substance is the nurse referring?

endorphins The nurse is referring to the release of endorphins, which are natural analgesic substances released by the movement of the client on the birth ball. The nurse should encourage the client to rock or sit on the birth ball. This causes the release of endorphins. The client's movement on the birth ball does not produce prostaglandins, progesterone, or relaxin. Prostaglandins are local hormones that bring about smooth muscle contractions in the uterus. Progesterone is a hormone involved in maintaining pregnancy. Relaxin is a hormone that causes backache during pregnancy by acting on the pelvic joints. pg 468

When planning the care of a woman in the active phase of labor, the nurse would anticipate assessing the fetal heart rate at which interval?

every 15 to 30 minutes During the active phase of labor, FHR is monitored every 15 to 30 minutes. FHR is assessed every 30 to 60 minutes during the latent phase of labor. The woman's temperature is typically assessed every 4 hours during the first stage of labor and every 2 hours after ruptured membranes. Blood pressure, pulse, and respirations are assessed every hour during the latent phase and every 30 minutes during the active and transition phases. Contractions are assessed every 30 to 60 minutes during the latent phase, every 15 to 30 minutes during the active phase, and every 15 minutes during transition.

The nurse is assessing the laboring client to determine fetal oxygenation status. What indirect assessment method will the nurse likely use?

external electronic fetal monitoring Analysis of the FHR using external electronic fetal monitoring is one of the primary evaluation tools used to determine fetal oxygen status indirectly. Fetal pulse oximetry measures fetal oxygen saturation directly and in real time. It is used with electronic fetal monitoring as an adjunct method of assessment when the FHR pattern is abnormal or inconclusive. Fetal scalp blood is obtained to measure the pH. The fetal position can be determined through ultrasonography or abdominal palpation but is not indicative of fetal oxygenation.

The nurse is assessing the laboring client to determine fetal oxygenation status. What indirect assessment method will the nurse likely use?

external electronic fetal monitoring Analysis of the FHR using external electronic fetal monitoring is one of the primary evaluation tools used to determine fetal oxygen status indirectly. Fetal pulse oximetry measures fetal oxygen saturation directly and in real time. It is used with electronic fetal monitoring as an adjunct method of assessment when the FHR pattern is abnormal or inconclusive. Fetal scalp blood is obtained to measure the pH. The fetal position can be determined through ultrasonography or abdominal palpation but is not indicative of fetal oxygenation. pg 455

As a woman enters the second stage of labor, which would the nurse expect to assess?

feelings of being frightened by the change in contractions

As a woman enters the second stage of labor, which would the nurse expect to assess?

feelings of being frightened by the change in contractions The nature of contractions changes so drastically to an urge to push that this can be frightening. pg 487

A pregnant client in her 32nd week of gestation has been admitted to a health care center reporting decreased fetal movement. What should the nurse determine first before placing the fetoscope on the woman's abdomen, so as to auscultate the fetal heart sounds?

fetal back The nurse assessing the client should first determine the fetal back before placing the fetoscope on the client's abdomen. The fetal back is determined first because it is through the back that the heart signals are best transmitted. During labor, the fetal heart rate should be assessed to check for any variations indicating distress. Fetal heart rate is auscultated by placing a fetoscope on the client's abdomen in the area of the fetal back. Determining the fetal head, shoulders, and the buttocks would be of no help in localizing the heart sounds.

A pregnant client in her 32nd week of gestation has been admitted to a health care center reporting decreased fetal movement. What should the nurse determine first before placing the fetoscope on the woman's abdomen, so as to auscultate the fetal heart sounds?

fetal back The nurse assessing the client should first determine the fetal back before placing the fetoscope on the client's abdomen. The fetal back is determined first because it is through the back that the heart signals are best transmitted. During labor, the fetal heart rate should be assessed to check for any variations indicating distress. Fetal heart rate is auscultated by placing a fetoscope on the client's abdomen in the area of the fetal back. Determining the fetal head, shoulders, and the buttocks would be of no help in localizing the heart sounds. pg 453

If a fetus were not receiving enough oxygen during labor because of uteroplacental insufficiency, which pattern would the nurse anticipate seeing on the monitor?

fetal heart rate declining late with contractions and remaining depressed Lack of blood supply to the fetus because of poor placental filling prevents the fetal heart rate from recovering immediately following a contraction.

A nurse is monitoring a fetal heart rate (FHR) pattern on her client in labor. The earlier baseline FHR was 140. The FHR now is 168. The nurse knows that which factors can affect changes in the FHR? Select all that apply.

fetal movement fetal distress maternal fever An increase in the FHR from baseline can mean that there is some type of fetal distress. This can happen with a maternal fever also. Narcotics may affect the heart rate variability but not the baseline FHR. Fetal movement can cause an increase in FHR, and utero-placental insufficiency can cause late decelerations in the FHR.

The nurse is admitting a client who is in early labor. After determining that the birth is not imminent, which assessment should the nurse perform next?

fetal status

when does the fetal membrane typically rupture?

first stage of labor

A pregnant client with a history of spinal injury is being prepared for a cesarean birth. Which method of anesthesia is to be administered to the client?

general anesthesia General anesthesia is administered in emergency cesarean births. Local anesthetic is injected into the superficial perineal nerves to numb the perineal area generally before an episiotomy. Although an epidural block is used in cesarean births, it is contraindicated in clients with spinal injury. Regional anesthesia is contraindicated in cesarean births.

A client in active labor is given spinal anesthesia. Which information would the nurse include when discussing with the client and family about the disadvantages of spinal anesthesia?

headache following anesthesia The nurse should inform the client and her family about the possibility of headache after spinal anesthesia. The drug is retained in the mother's body and not passed to the fetus. There may be uterine atony, and not excessive uterine contractions, following spinal anesthesia. Spinal anesthesia may lead to bladder atony, and not an increased frequency of micturition.

In providing culturally competent care to a laboring woman, which is a priority?

identify how the client expresses labor pain.

The nurse is assisting a client through labor, monitoring her closely, now that she has received an epidural. The nurse would report which finding to the anesthesiologist?

inability to push If she is not able to push, her epidural dose may be adjusted to decrease the impact on the sensory system. Dry lips indicate that she may need fluids, so the nurse should give her some ice chips or a drink of water. Urinary retention and rapidly progressing labor should be directly reported to the obstetrician, not the anesthesiologist.

When teaching the pregnant patient about self-medicating for pain during labor, why did the nurse instruct the patient to avoid taking acetylsalicylic acid?

interference with blood coagulation with increased risk of bleeding in mother or infant.

What is an epidural?

is a numbing medicine given by injection (shot) in the back. It numbs or causes a loss of feeling in the lower half your body. This lessens the pain of contractions during childbirth

A nurse recommends to a client in labor to try concentrating intently on a photo of her family as a means of managing pain. The woman looks skeptical and asks, "How would that stop my pain?" Which explanation should the nurse give?

it distracts you from the sensations of pain.

At which time during a woman's labor might the nurse assist with a pudendal block?

just before birth Pudendal block is a local block in the perineal area and is used to numb for birth. Application before labor begins or while it is in the early stages would be counterproductive, as the client would not have proper feeling and would have a harder time pushing. After birth it is pointless; the most painful part is over

A patient is admitted to the labor and delivery unit. Upon examination, she is found to be dilated 3 cm. The nurse notes that the woman is having contractions that last about 45 seconds and are about 5 minutes apart. Based on this information, in which phase of labor is this patient?

latent phase Contractions during the latent phase of labor are typically 5 to 10 minutes apart and last 30 to 45 seconds. The cervix is dilated 1 to 3 cm, and effacement begins.

The student nurse is preparing to assess the fetal heart rate (FHR). She has determined that the fetal back is located toward the client's left side, the small parts toward the right side, and there is a vertex (occiput) presentation. The nurse should initially begin auscultation of the fetal heart rate in the mother's:

left lower quadrant. The best position to auscultate fetal heart tones in on the fetus back. In this position, the best place for the FHR monitor is on the left lower quadrant.

The student nurse is preparing to assess the fetal heart rate (FHR). She has determined that the fetal back is located toward the client's left side, the small parts toward the right side, and there is a vertex (occiput) presentation. The nurse should initially begin auscultation of the fetal heart rate in the mother's:

left lower quadrant. The best position to auscultate fetal heart tones in on the fetus back. In this position, the best place for the FHR monitor is on the left lower quadrant. pg 455

A woman in labor who is receiving an opioid for pain relief is to receive promethazine. The nurse determines that this drug is effective when the woman demonstrates which finding?

less anxiety Promethazine is used in combination with an opioid to decrease nausea and vomiting and lessen anxiety. It may also be used to increase sedation. It does not affect the progress of labor. Benzodiazepines are used to calm a woman who is out of control, allowing her to relax enough to participate effectively during labor.

A nurse is required to obtain the fetal heart rate (FHR) for a pregnant client. If the presentation is cephalic, which maternal site should the nurse monitor to hear the FHR clearly?

lower quadrant of the maternal abdomen In a cephalic presentation, the FHR is best heard in the lower quadrant of the maternal abdomen. In a breech presentation, it is heard at or above the level of the maternal umbilicus.

Adverse effects of an epidural

maternal hypotension, fetal bradycardia, inability to feel the urge to void, loss of the bearing down reflex, prolonged 2nd stage, catheter migration (respiratory distress if it goes up), maternal fever, hot spots, bladder distention

green amniotic fluid

may indicate that the fetus has passed meconium secondary to transient hypoxia, prolonged pregnancy, cord compression, intrauterine growth restriction (IUGR), maternal hypertension, diabetes, or chorioamnionitis normal if baby is in breech position = meconium

The nurse notes that a client's amniotic fluid is green when the membranes rupture. What finding would the nurse document?

meconium in the amniotic sac Green tinted fluid with ROM is indicative of meconium in the amniotic sac, or the infant having a bowel movement in utero. Infection would be shown by pus or cloudy fluid. Umbilical cord prolapse occurs when pressure on the cord stops the flow of oxygen to the fetus. Amniotic embolism results when amniotic fluid enters circulation.

The nurse is assessing a client in labor for pain and notes she is currently not doing well handling the increased pain. Which opioid can the nurse offer to the client to assist with pain control?

meperidine

leopold maneuveurs

method for determining the presentation, position, and lie of the fetus through the use of four specific steps. -a longtiudinal lie is expected -position can be cephalic, breech, or shoulder. Maneuver 1: What fetal part (head or buttocks) is located in the fundus (top of the uterus)? Maneuver 2: On which maternal side is the fetal back located? (Fetal heart tones are best auscultated through the back of the fetus.) Maneuver 3: What is the presenting part? Maneuver 4: Is the fetal head flexed and engaged in the pelvis?

goal of FHR monitoring

mortality and morbidity by ensuring that all fetal hypoxic insults are identified in time to allow removal or alteration of the reason for them, or to enable a safe birth of the fetus before irreversible asphyxia damage occurs

second manuever

move hands down the lateral sides of the abdomen to palpate on which side the back is located (feels hard and smooth). continue to determine where limbs are locates

Cell Transport

movement of molecules across a cell membrane from an area of lower concentration to one of higher concentration

Chemical elements that make up carbohydrates

oxygen, carbon, hydrogen

When assessing fetal heart rate patterns, which finding would alert the nurse to a possible problem?

prolonged decelerations Prolonged decelerations are associated with prolonged cord compression, abruptio placentae, cord prolapse, supine maternal position, maternal seizures, regional anesthesia, or uterine rupture. Variable decelerations are the most common deceleration pattern found. They are usually transient and correctable. Early decelerations are thought to be the result of fetal head compression. They are not indicative of fetal distress and do not require intervention. Fetal accelerations are transitory increases in FHR and provide evidence of fetal well-being.

A woman is in the fourth stage of labor. During the first hour of this stage, the nurse would assess the woman's fundus at which frequency?

q15min

A nurse is caring for a client who has been administered an epidural block. Which should the nurse assess next?

respiratory rate The nurse must monitor for respiratory depression. Monitoring the client's respiratory rate will be the best indicator of respiratory depression.

RNA

single-stranded nucleic acid that contains the sugar ribose

Forms of matter

solid, liquid, gas

A pregnant client requires administration of an epidural block for management of pain during labor. For which conditions should the nurse check the client before administering the epidural block? Select all that apply.

spinal abnormality hypovolemia coagulation defects The nurse should check for any abnormality of the spine, hypovolemia, or coagulation defects in the client. An epidural is contraindicated in women with these conditions. Varicose veins and skin rashes or bruises are not contraindications for an epidural block. They are contraindications for massage used for pain relief during labor.

A nurse is providing care to a woman during the third stage of labor. Which finding would alert the nurse that the placenta is separating?

sudden gush of dark blood from the vagina Signs that the placenta is separating including a firmly contracting uterus, a change in uterine shape from discoid to globular ovoid, a sudden gush of dark blood from the vaginal opening, and lengthening of the umbilical cord protruding from the vagina.

While waiting for the placenta to deliver during the third stage of labor the nurse must assess the new mother's vital signs every 15 minutes. What sign would indicate impending shock?

tachycardia and a falling blood pressure Monitor the woman's vital signs at least every 15 minutes during the third stage of labor. Tachycardia and a falling blood pressure are signs of impending shock; the nurse should immediately report these signs. pg 491

A woman presents in advanced labor, and birth appears imminent. What is the most important and appropriate aspect of admission for this woman?

taking her blood pressure and determining whether clonus or edema is present In advanced labor the most important assessments must be completed first. The assessment for signs or symptoms of preeclampsia must be assessed first. The history can be obtained after the birth of the baby or if labor slows down. Plans for the newborn can be figured out later. Blood tests can be run as soon as a sample can be taken from the mother.

A primigravida client has just arrived in early labor and is showing signs of extreme anxiety over the birthing process. Why should the nurse prioritize helping the client relax?

Anxiety can slow down labor and decrease oxygen to the fetus.

A woman's husband expresses concern about risk of paralysis from an epidural block being given to his wife. Which would be the most appropriate response by the nurse?

"The injection is given in the space outside the spinal cord."

The spouse of a pregnant patient is concerned about the risk of paralysis from an epidural block being used during labor. How should the nurse respond to the spouse's concern?

"The injection is given in the space outside the spinal cord."

A woman's husband expresses concern about risk of paralysis from an epidural block being given to his wife. Which would be the most appropriate response by the nurse?

"The injection is given in the space outside the spinal cord." An epidural block, as the name implies, does not enter the spinal cord but only the epidural space outside the cord.pg 474

Amniotomy

Artificial rupture of the membranes by an amnihook.

A woman asks the nurse if she can eat something during labor. Which response by the nurse would be best?

"You could have some hard candy to suck on." If women are kept NPO during labor, they can be administered anesthesia safely in an emergency. Stomach-emptying time is decreased.

A client asks her nurse what effleurage means. After instruction is given, the nurse determines learning has taken place when the client states:

"Effleurage is light abdominal massage used to displace pain." Effleurage is a light abdominal massage used to keep the laboring woman's focus on the massage instead of the pain of labor.

A client asks her nurse what effleurage means. After instruction is given, the nurse determines learning has taken place when the client states:

"Effleurage is light abdominal massage used to displace pain." Effleurage is a light abdominal massage used to keep the laboring woman's focus on the massage instead of the pain of labor. pg 470

A woman refuses to have an epidural block because she does not want to have a spinal headache after birth. What would be the nurse's best response?

"Spinal headache is not a usual complication of epidural blocks." Because epidural anesthesia does not enter the cerebral spinal fluid space, it is unlikely to cause a "spinal headache."

The health care provider approves a labor plan which includes analgesia. The client questions how analgesia will help her pain during labor. Which answer is best?

"The analgesia will reduce the sensation of pain for a limited period of time."

When a client is counseled about the advantages of epidural anesthesia, which statement made by the counselor would indicate the need for further teaching?

"You have no trouble walking around and using the bathroom after you receive the epidural." Epidural anesthesia impairs mobility; most clients are placed on bed rest after epidural anesthesia is given. Urinary catheterization is frequently required.

The health care provider is evaluating a high-risk woman for a continuous internal monitoring. It would be most appropriate to meet which criterion?

"Your labor signs have not changed; we are looking for changes in your labor pattern before we check you again." The cervix must be assessed with a vaginal exam. The frequency of vaginal exams is based on the signs of changes in labor. The client has not demonstrated any changes in her labor pattern; the nurse should provide education on the reason for not checking her. pg 451

A woman at 39 weeks gestation has been in labor for 8 hours and is asking how far she is dilated. She attended childbirth classes and is aware of the stages and phases of labor. She had a vaginal exam 30 minutes prior to her asking again. How should the nurse respond to her question?

"Your labor signs have not changed; we are looking for changes in your labor pattern before we check you again." "Your labor signs have not changed; we are looking for changes in your labor pattern before we check you again." The cervix must be assessed with a vaginal exam. The frequency of vaginal exams is based on the signs of changes in labor. The client has not demonstrated any changes in her labor pattern; the nurse should provide education on the reason for not checking her.

A low-risk client is in the active phase of labor. The nurse evaluates the fetal monitor strip at 10:00 a.m. and notes the following: moderate variability, FHR in the 130s, occasional accelerations, and no decelerations. At what time should the nurse reevaluate the FHR?

1030

A client is in the active phase of labor. She is a low-risk client. The nurse evaluates the fetal monitor strip at 10:00 a.m. Moderate variability is present. The FHR is in the 130s with occasional accelerations, no decelerations. At what time does the nurse need to reevaluate the FHR?

10:30 a.m. Assess and document fetal status at least every 30 minutes. Record the baseline FHR every 30 minutes and evaluate the fetal monitor tracing for abnormal patterns. Variability should be present, except for brief periods of fetal sleep or when the mother receives narcotics or other selected medications, and no late decelerations should be present. Accelerations of the FHR are normal.

Normal fetal heart rate

110-160 bpm

What is the normally accepted fetal heart rate range?

110-160 bpm The standard acceptable fetal heart rate baseline is the range of 110-160 beats per minute. Sustained heart rates above or below the norm are cause for concern.

Fetal heart rate monitoring reveals baseline tachycardia in the fetus. Which rate would be most likely?

164 beats per minute A fetal heart rate of 164 beats per minute (bpm) indicates fetal tachycardia. The normal range of FHR is between 120 and 160 bpm. When the FHR is above 160 bpm, it should be considered as fetal tachycardia. Therefore, a FHR of 164 beats per minute is considered tachycardia.

pH of amniotic fluid

6.5-7.5

initiates cervical dilation

30 mmHG or higher

The nurse tests the pH of fluid found on the vaginal exam and determines that the woman's membranes have ruptured based on which result?

6.5 Amniotic fluid is alkaline, so the membranes are probably ruptured if the pH ranges from 6.5 to 7.5.

The nurse is caring for a client in active labor who has had a fetal blood sampling to check for fetal hypoxia. The nurse determines that the fetus has acidosis when the pH is:

7.15 or less.

The nurse is caring for a client in active labor who has had a fetal blood sampling to check for fetal hypoxia. The nurse determines that the fetus has acidosis when the pH is:

7.15 or less. In the hypoxic fetus, the pH will fall below 7.2, which is indicative of fetal distress. pg 464

Butorphanol (Stadol)

Given IV Q 2-4 hr. Rapidly transferred across the placenta. Causes neonatal respiratory depression.

Systems and functions

Integumentary system: protects body; helps regulate body temperature; eliminates some wastes; helps make vitamin D; detects sensations such as touch, pain, warmth, and cold; stores fat and provides insulation Skeletal system: supports and protects body; provides surface area for muscle attachments; aids body movements; houses cells that produces blood cells; stores minerals and lipids Muscular system: participates in body movements, such as walking; maintains posture; produces heat Nervous system: generates action potentials (nerve impulses) to regulate body activities; detects changes in body's internal and external environments, interprets changes, and responds by causing muscular contractions or glandular secretions Endocrine system: regulates body activities by releasing hormones (chemical messengers transported in blood from endocrine gland or tissue to target organ) Cardiovascular system: heart pumps blood through blood vessels; blood carries oxygen and nutrients to cells and carbon dioxide and wastes away from cells and helps regulate acid-base balance, temperature, and water content of body fluids; blood components help defend against disease and repair damaged blood vessels. Lymphatic system: returns proteins and fluids to blood; carries lipids from gastrointestinal tract to blood; contains sites of maturation and proliferation of B cells and T cells that protect against disease-causing microbes Respiratory system: transfers oxygen from inhaled air to blood and carbon dioxide from blood to exhaled air; helps regulate acid- base balance of body fluids; air flowing out of lungs through vocal cords produces sounds Digestive system: achieves physical and chemical breakdown of food; absorbs nutrients; eliminates solid wastes Urinary system: produces, stores, and eliminates urine; eliminates wastes and regulates volume and chemical composition of blood; helps maintain the acid-base balance of body fluids; maintains body's mineral balance; helps regulate production of red blood cells Reproductive system: gonads produce gametes (sperm or oocytes) that unite to form a new organism; gonads also release hormones that regulate reproduction and other body processes; associated organs transport and store gametes; mammary glands produce milk

There are advantages and disadvantages to any kind of method used to control pain during labor and birth. What is an advantage of opioid administration?

It can be administered by the nurse.

General anesthesia is not used frequently in obstetrics because of the risks involved. There are physiologic changes that occur during pregnancy that make the risks of general anesthesia higher than it is in the general population. What is one of those risks?

Neonatal depression is possible

A woman in labor at the hospital has just received an epidural block. Which intervention is a priority before and during epidural placement?

Provide adequate IV fluids to maintain her blood pressure. The client will need to have a bolus of IV fluids prior to and then maintained during the epidural to be prepared in the event of the hypotensive episodes that may accompany epidural placement. The hypotensive event is transitory, and increasing oral hydration is unnecessary and may lead to nausea later. Monitor the mother's body temperature, but wait for instructions from the primary care provider as to when to administer medication. Bradycardia is not a common side effect of epidural medication. pg 474

A nurse is caring for a client who has been administered an epidural block. Which should the nurse assess next?

respiratory rate The nurse must monitor for respiratory depression. Monitoring the client's respiratory rate will be the best indicator of respiratory depression. pg 474

t or f: If the hands do not move together and stop because of resistance, the fetal head is engaged into the woman's pelvic inlet

true

The nurse is monitoring a client's uterine contractions. Which factors should the nurse assess to monitor uterine contraction? Select all that apply.

uterine resting tone frequency of contractions intensity of contractions The nurse should assess the frequency of contractions, intensity of contractions, and uterine resting tone to monitor uterine contractions. Monitoring changes in temperature and blood pressure is part of the general physical examination and does not help to monitor uterine contraction.

To assess the frequency of a woman's labor contractions, the nurse would time:

the beginning of one contraction to the beginning of the next.

Diazepam (Valium)

*2-5mg IV* Is given to enhance pain relief of opioid and cause sedation. May be used to stop eclamptic seizures Decreases N/V Can cause newborn depression; therefore, lowest possible dose should be used.

Prochlorperazine (Compazine)

*5-10mg IV or IM* Frequently given with morphine sulfate for sleep during prolonged latent phase; counteracts the nausea that opioids can produce.

Carbohydrates

the starches and sugars present in foods

The nurse is caring for a client who has been in labor for the past 8 hours. The nurse determines that the client has transitioned into the second stage of labor based on which sign?

the urge to push occurs

A nurse is preparing a patient for rhythm strip testing. She places the woman into a semi-Fowler's position. What is the appropriate rationale for this measure?

to prevent supine hypotension syndrome

During labor, progressive fetal descent occurs. Place the stations listed in their proper sequence from first to last. All options must be used

-4 station -2 station 0 station +2 station +4 station Progressive fetal descent (-5 to +4) is the expected norm during labor, moving downward from the negative stations to zero station to the positive stations in a timely manner. pg 452

A nurse practitioner is conducting an in-service education program for a group of nurses working in the labor and birth unit. The program is focusing on interpreting FHR patterns. The nurse practitioner determines that the teaching was successful when the group identifies which patterns as indicating abnormal fetal acid-base status? Select all that apply -fetal bradycardia -fetal tachycardia -sinusoidal pattern -recurrent late decelerations

-fetal bradycardia -sinusoidal pattern -recurrent late decelerations

Which statement is true regarding analgesia versus anesthesia?

Analgesia only reduces pain, but anesthesia partially or totally blocks all pain in a particular area.

Leopold maneuver sequence

Palpate the fundus, determine location of fetal back, palpate fetal part presenting at the inlet, palpate the cephalic prominence to identify attitude of head.

A primigravida client has just arrived in early labor and is showing signs of extreme anxiety over the birthing process. Why should the nurse prioritize helping the client relax?

Anxiety can slow down labor and decrease oxygen to the fetus

During an admission assessment of a client in labor, the nurse observes that there is no vaginal bleeding yet. What nursing intervention is appropriate in the absence of vaginal bleeding when the client is in the early stage of labor?

Assess amount of cervical dilation. If vaginal bleeding is absent during admission assessment, the nurse should perform vaginal examination to assess the amount of cervical dilation. Hydration status is monitored as part of the physical examination. A urine specimen is obtained for urinalysis to obtain a baseline. Vital signs are monitored frequently throughout the maternal assessment. pg 451

As a woman enters the second stage of labor, her membranes spontaneously rupture. When this occurs, what would the nurse do next?

Assess fetal heart rate for fetal safety.

The laboring client is on continuous fetal monitoring when the nurse notes a decrease in the fetal heart rate with variable deceleration to 75 bpm. What is the initial nursing intervention?

Change the position of the client. Variable decelerations often indicate a type of cord compression. The initial response is to change the position and try to release the cord compression. If this does not work, apply oxygen while using the call light to alert others. If this continues, her fluid status needs to be assessed before increasing her IV rate.

A 39-week-gestation client presents to the labor and birth unit reporting abdominal pain. What should the nurse do first?

Determine if the client is in true or false labor. When a nurse first comes in contact with a pregnant client, it is important to first ascertain whether the woman is in true or false labor. Information regarding the number of pregnancies or history of drug allergy is not important criteria for admitting the client. The healthcare provider should be notified once the nurse knows the client's current status. pg 478

The nurse is monitoring a client in the first stage of labor. The nurse determines the client's uterine contractions are effective and progressing well based on which finding?

Dilation of cervix

Transcutaneous electrical nerve stimulation (TENS) reduces pain by which of the following mechanisms?

Electrical impulses are created that interfere with nerve transmission.

Branches of Anatomy

Embryology, developmental biology, cell biology, histology, gross anatomy, systemic anatomy, regional anatomy, surface anatomy, imaging anatomy, pathological anatomy

The nurse is assisting a client through labor, monitoring her closely now that she has received an epidural. Which finding should the nurse prioritize to the anesthesiologist?

Inability to push

5 P's

Pelvis, passenger, power (dilation/effacement/station), position (maternal), psychological response

A nurse is caring for a female client in labor who has chosen hydrotherapy as her pain management for labor. As the nurse prepares the client for this treatment, which procedure is recommended as the most appropriate consideration?

The client should be in active labor. Most recommendations for hydrotherapy, or water therapy, include active labor. If the client is not in active labor, the contractions could slow because of relaxation of muscles. There is no time limit for water therapy; it is provided for comfort. The water temperature should not be higher than the maternal body temperature, and water therapy can be used with intact or ruptured membranes.

To assess the frequency of a woman's labor contractions, the nurse would time:

the beginning of one contraction to the beginning of the next. Measuring from the beginning of one contraction to the next marks the time between contractions. pg 453

On examination, the nurse determines the client is at 50% effacement. This means:

the cervical canal is 1 cm long. Effacement refers to the length of the cervical canal. At 0%, the cervical canal is 2 cm long; at 50%, 1 cm long; and at 100%, the cervical canal is obliterated.

A woman's perception of pain can differ according to all of the following except:

the presentation, lie, and attitude of the fetus. Fetal position can influence a client's perception of pain. Fetal attitude does not influence a client's perception of pain.

Diffusion

the process by which molecules move from an area of higher concentration to an area of lower concentration

Chemical reaction

the process by which one or more substances change to produce one or more different substances

Cell membrane

the semipermeable membrane surrounding the cytoplasm of a cell.

Meperidine (Demerol)

*25-75mg IV* Can cause CNS depression Decreases fetal variability

A nurse places an external fetal monitor on a woman in labor. Which instruction would be best to give her?

Lie on her side so she is comfortable. The best position for all women during labor is on their side.

Solution

A mixture that forms when one substance dissolves another.

A client states that "she thinks" her water has broken. Which best provides confirmation of the rupture of membranes?

A positive Nitrazine test

Fern Test

A tests used if the Nitrazine swab was inconclusive. The sample is obtained and looked at through a microscope. Look for fern pattern which indicates the presence of amniotic fluid.

As a woman enters the second stage of labor, her membranes spontaneously rupture. When this occurs, what would the nurse do next?

Assess fetal heart rate for fetal safety. Rupture of the membranes may lead to a prolapsed cord. Assessment of FHR detects this.

contraction pattern every 15 minutes

Assess the contraction pattern every 15 minutes. The pattern will be similar to that found in the transition phase (i.e., contractions occur every two to three minutes, last 60 to 90 seconds, and are of strong intensity).

A client has just had an epidural placed. Before the procedure, her vital signs were as follows: BP 120/70, P90 bmp, R18 per min, and O2 sat 98%. Now, 3 minutes after the procedure, the client says she feels light-headed and nauseous. Her vital signs are BP 80/40, P100 bmp, R20 per min, and O2 sat 96%. Which interventions should the nurse perform?

Assist the client to semi-Fowler's position, assess the fetal heart rate, start an IV bolus of 500 mL, and administer oxygen via face mask. In a pregnant woman, hypotension is best managed in the left lateral or semi-Fowler's position owing to risk of supine hypotension in the supine position and in Trendelenburg's position. The sitting position could exacerbate hypotension. Naloxone is administered for respiratory depression. When the mother experiences a change in vital signs, this may affect the fetal heart rate.

The following are nursing measures commonly offered to women in labor. Which nursing intervention probably would be most effective in applying the gate-control theory for relief of labor pain?

Massage the woman's back. Gate control is based on the idea of distraction or redirection of the conduction of impulses up the neural pathways. Massage redirects the paths of sensation away from the pain to the other area.

A client has presented in the early phase of labor, experiencing abdominal pain and signs of growing anxiety about the pain. Which pain management technique should the nurse prioritize at this stage?

Practicing effleurage on the abdomen

The nurse is preparing to administer an intradermal water injection to a client who is in labor. Which action should the nurse prioritize?

Prepare four 1 mL syringes of 0.05 to 0.1 mL sterile water using a 25 gauge needle.

Elements of an atom

Proton: Positive, in nucleus Neutron: Neutral, in nucleus Nucleus: center consisting of protons and neutrons Electrons: Tiny and Negative, revolve around nucleus

The nursing instructor is teaching a group of nursing students about the various responsibilities of the labor and delivery medical team. The instructor determines the session is successful when the students correctly choose which function as the primary role of the LPN/LVN members of the team?

Provide care under the supervision of an RN.

A woman arrives in the labor and birth department and is panting and screaming "the baby is coming". What is the priority intervention by the nurse?

Quickly move the woman to a labor bed, and check the perineum. The woman is showing signs of advanced labor, possibly in transition or stage two. She needs to be managed as an imminent birth and taken directly to a room for vaginal assessment. Vital signs, medical/obstetrical history, and her room assignment can be taken care of later in the process.

Fetal position

Relationship of the presenting part to the maternal pelvis Written occiput or mentrum for cephalic, scapular acromion process (A) for breech

What is amnioinfusion?

Technique where warmed sterile saline or LR is introduced into the uterus via an intrauterine pressure catheter. Used intrapartally to increase the volume of fluid in oligohydramnios to alleviate cord compression, or to dilute moderate to heavy meconium released in utero by a stressed fetus.

A client has just arrived at the hospital, in early labor, showing signs of extreme anxiety over the birth to come. Why is it so important that the nurse help the client relax?

The client's anxiety can actually slow down the labor process and decrease the amount of oxygen reaching the uterus and the fetus. Anxiety out of control can decrease the oxygen of the mother by increasing her respiratory rate and increasing the demand on her body, and have a negative impact on the fetus. Encourage control of the anxiety. Anxiety will not negatively affect the action of the epidural. It is premature to be stern with the client. While it is preferable that she save her energy, it is not damaging to her or to the fetus if she does not sleep. pg464

The nurse instructs the client about skin massage and the gate-control theory of pain. Which statement would be appropriate for the nurse to include for client understanding of the nonpharmacologic pain relief methods?

This is a technique to prevent the painful stimuli from entering the brain.

Purpose of doing a vaginal exam during labor

The purpose of performing a vaginal examination is to assess the amount of cervical dilation, the percentage of cervical effacement, and the fetal membrane status and to gather information on presentation, position, station, degree of fetal head flexion, and presence of fetal skull swelling or molding

Which signs signify that the second stage of labor has begun?

The urge to push occurs. Second stage of labor is the pushing stage; this is typically identified by the woman's urge to push or a feeling of needing to have a bowel movement. The emotional state may be altered due to pain and pressure. Contraction frequency is variable and not clearly indicative of a particular stage. The fetus can be at stage -1 for any length of time.

The nurse is aware that cord compression is not continuous when variable decelerations occur and that compression happens when which of the following takes place?

The uterus contracts and squeezes the cord against the fetus. Cord compression is not continuous when variable decelerations are occurring. The compression occurs when the uterus contracts and squeezes the cord against the fetus. It is relieved when the uterus relaxes between contractions. Prematurity and fetal sleep will cause decreased or absent variabilty.

The laboring client who is at 3 cm dilation and 25% effaced is asking for analgesia. The nurse explains the analgesia usually is not administered prior to the establishment of the active phase. What is the appropriate rationale for this practice?

This may prolong labor and increase complications.

What is the most important thing a nurse can do during labor and birth to prevent maternal and fetal infection?

Thoroughly wash the hands before and after client contact. The most important infection control technique in any health care setting is thoroughly washing hands on routine basis. Keeping the area clean is secondary, but is also important.

A nurse is preparing a patient for rhythm strip testing. She places the woman into a semi-Fowler's position. What is the appropriate rationale for this measure?

To prevent supine hypotension syndrome The term "rhythm strip testing" means assessment of the fetal heart rate for whether a good baseline rate and long- and short-term variability are present. For this, help a woman into a semi-Fowler's position (either in a comfortable lounge chair or on an examining table or bed with an elevated backrest) to prevent her uterus from compressing the vena cava and causing supine hypotension syndrome during the test. Placing her in this position does not decrease the heart rate of the fetus. It is not done to aid the woman as she pushes in labor, as she is not in labor yet. It is not done to prevent her from falling out of bed.

If the monitor pattern of uteroplacental insufficiency were present, which action would the nurse do first?

Turn her or ask her to turn to her side

Fetal presentation

What's coming to the outlet first - cephalic or breech

Cell Structure and Function

cells are small but highly organized, they contain specialized structures that carry out the cell's life processes

Which intervention has been demonstrated to reduce the release of catecholamines and anxiety, and has resulted in better birth outcomes for women?

continuous labor support Continuous labor support has been demonstrated to result in better labor outcomes in the current research. Continuous labor support is defined as caring for the labor patient by a nurse, doula, or primary support person who does not leave the client. Massage therapy is effective in that it engages gate control. Pharmacological interventions are useful but pose potential side effects to the mother and fetus. Hypnosis is less well understood.

A client is now in the second stage of labor. While doing the assessment, the nurse would gather what data at this time?

contraction pattern every 15 minutes Assess the contraction pattern every 15 minutes. The pattern will be similar to that found in the transition phase (i.e., contractions occur every two to three minutes, last 60 to 90 seconds, and are of strong intensity).

A client is now in the second stage of labor. While doing the assessment, the nurse would gather what data at this time?

contraction pattern every 15min.

A nurse is auscultating the fetal heart rate of a woman in labor. To ensure that the nurse is assessing the FHR and not the mother's heart rate, which action would be most appropriate for the nurse to do?

palpate the mothers radial pulse at the same time

The nurse explains Leopold's maneuvers to a pregnant client. For which purposes are these maneuvers performed? Select all that apply.

determining the presentation of the fetus determining the position of the fetus determining the lie of the fetus Leopold maneuvers help the nurse to determine the presentation, position, and lie of the fetus. The approximate weight and size of the fetus can be determined with ultrasound sonography or abdominal palpation.

When assessing a woman in the first stage of labor, the nurse recognizes that the most conclusive assessment that uterine contractions are effective would be:

dilatation of cervix. The best determination of effective contractions is dilation of the cervix. Engagement, membrane rupture, and bloody show may all occur before the cervix has dilated.

A woman is lightly stroking her abdomen in rhythm with her breathing during contractions. The nurse identifies this technique as:

effleurage. Effleurage is a light, stroking, superficial touch of the abdomen in rhythm with breathing during contractions. Acupressure involves the application of a finger or massage at a trigger point to reduce the pain sensation. Patterned breathing involves controlled breathing techniques to reduce pain through a stimulus-response conditioning. Therapeutic touch involves light or firm touch to the energy field of the body using the hands to redirect the energy fields that lead to pain. pg 470

Which intervention would be least effective in caring for a woman who is in the transition phase of labor?

encouraging the woman to ambulate Although ambulating is beneficial during early and possibly even active labor, the strong and frequent contractions experienced and the urge to bear down may make ambulating quite difficult. During transition, women should continue to breathe with contractions and focus on one contraction at a time. Providing one-to-one support at this time helps the woman cope with the events of this phase, as well as help her maintain a sense of control over the situation. pg 481

The nurse is caring for a client who is considered low-risk and in active labor. During the second stage, the nurse would evaluate the client's FHR at which frequency?

every 15 minutes It is recommended that the FHR be assessed during the second stage of labor every 15 minutes for the low-risk woman and every 5 minutes for the high-risk woman and during the pushing stage.

A patient who is in her 9th month of pregnancy comes to the emergency department and reports that bright red blood is coming from her vagina. She denies having any pain. What needs to be ruled out before a vaginal examination can be performed?

placenta previa Vaginal examinations should never be done if the woman presents with bright red painless bleeding until placenta previa is ruled out. The other options would not be concerns at this time based on the findings.


Ensembles d'études connexes

bio final chapters 52-56, chapter quizzes, old exams

View Set

med term - chp 15 - comp terms nbfwp

View Set

Chapter 16 New Motors (EV, Natural Gas, and Hydrogen)-combined

View Set

Characteristics of Bright Romanticism, Dark Romanticism, and Transcendentalism

View Set

Foundations of Nursing, Chapter 1 Nursing Today Review Questions

View Set

Chapter 2 - Financial Reporting:Its Conceptual Framework

View Set

Chapter 13 peds sensory and neurological disorders

View Set

Communication Skills - True False

View Set

Chapter 1: Fuel Metabolism and Nutrition - Basic Principles

View Set